Download ATLS EXAM PRE AND POST EXAM 2024-2025 and more Exams Nursing in PDF only on Docsity! 1 | P a g e ATLS EXAM PRE AND POST EXAM 2024-2025 WITH ACTUAL CORRECT QUESTIONS AND VERIFIED DETAILED ANSWERS |FREQUENTLY TESTED QUESTIONS AND SOLUTIONS |ALREADY GRADED A+|BRAND NEW!!!|GUARANTEED PASS |LATEST UPDATE A 24-year-old woman passenger in an automobile strikes the wind screen with her face during a head-on collision. In the ED, she is talking and has marked facial edema and crepitus. The highest priority should be given to: A. Lateral c-spine x-ray B. Upper airway protection C. Carotid pulse assessment D. Management of blod loss E. Determination of associated injuries B. Upper airway protection The driver of a single car crash is orotracheally intubated in the field by prehospital personnel after they identify a closed head injury and determine that the patient is unable to protect his airway. In the ED, the patient demonstrate decorticate posturing bilaterally. He is being ventilated with a bag-valve device, but his breath sounds are absent in the left hemithorax. His blood pressure is 160/80mmHg, heart rate is 70 beats per minute, and the pulse oximeter displays a hemoglobin oxygen saturation of 96%. The next step in assessing and managing this patient should be to: A. Determine the arterial blood gases B. Obtain a lateral cervical spine x-ray C. Assess placement of the endotracheal tube D. Perform needle decompression of the left chest E. Insert a thoracostomy in the left hemithorax. C. Assess placement of the endotracheal tube 2 | P a g e The response to catecholamines in an injured, hypovolemic pregnant woman can be expected to result in: A. Placental abruption B. Fetal hypoxia and distress C. Fetal/maternal dysrhytmia D. Improved uterine blood flow E. Increased maternal renal blood flow B. Fetal hypoxia and distress A 22-year-old man sustains a gunshot wound to the left chest and is transported to a small community hospital at which surgical capabilites are not available. In the ED, a chest tube is inserted and 700ml of blood is evacuted. The trauma center accepts the patient in transfer. Just before the patient is placed in an ambulance for transfer, his blood pressure decreases to 80/68mmHg and his heart rate increases to 136 beats per minute. The next step should be to: A. Clamp the chest tube B. Cancel the patients transfer C. Perform an ED thoracotomy D. Repeat the primary survey and proceed with transfer E. Delay the transfer until the referring doctor can contact a thoracic surgeon. D. Repeat the primary survey and proceed with transfer A young woman sustains a severe head injury as the result of a motor vehicular crash. In the ED, her GCS is 6. Her blood pressure is 140/90 mmHg and her heart rate 80 beats per minute. She is intubated and is being mechanically ventilated. Her pupils are 3mm in size and equally reactive to light. There is no other apparent injury. The most important principle to follow in early management of her head injury is to: A. Avoid hypotension B. Prevent secondary brain injury C. Agressively treat systemic hypertension D. Reduce meatbolic requirements of the brain E. Distinguish between intracranial hematoma and cerebral edema. A. Avoid hypotension A 6-year-old boy walking across the street is struck by the front bumper of a sports utility vehicle traveling at 32kph (20mph). Which one of the following statements is TRUE? A. A flail chest is probable B. A symptomatic cardiac contusion is expected C. A pulmonary contusion may be present in the absence of rib fractures D. Transection of the thoracic aorta is more likely than in an adult patient E. Rib fractures are commonly found in children with this mechanism of injury C. A pulmonary contusion may be present in the absence of rib fractures A 39-year-old man is admitted to the ED after an automobile collision. He is cyanotic, has insufficient respiratory effort, and has a GCS score of 6. His full beard makes it difficult to fit the oxygen facemask to his face. The most appropriate next step is to: 5 | P a g e D. Call the receiving hospital and speak to the surgeon on call E. Discuss the advisability of transfer with the patients family. D. Call the receiving hospital and speak to the surgeon on call During the third trimester of pregnancy, all of the following changes occur normally, EXCEPT a: A. Decrease in PaCO2 B. Decrease in the leukocyte count C. Reduce gastric emptying rate D. Diminished residual lung volume E. Diminished pelvic ligament tension B. Decrease in the leukocyte count In managing the head-injury patient, the most important initial step is to: A. Secure the airway B. Obtain c-spine film C. Support the circulation D. Control scalp hemorrhage E. Determine GCS score A. Secure the airway The first maneuver to improve oxygenation after chest injury is: A. Intubate the patient B. Assess arterial blood gases C. Administer supplemental oxygen D. Ascertain the need for a chest tube E. Obtain a chest x-ray C. Administer supplemental oxygen A 25-year-old man, injured in a motor vehicular crash, is admitted to the ED. His pupils react sluggishly and his eyes open to painful stimuli only. He does not follow commands, but he does moan periodically. His right arm is deformed and does not respond to painful stimulus; however, his left hand reaches purposefully toward the painful stimulus. Both legs are stiffly extended. His GCS score is: A. 7 B. 8 C. 9 D. 10 E. 11 9 A 20-year-old woman, at 32 weeks gestation, is stabbed in the upper right chest. In the ED, her blood pressure is 80/60mmHg. She is gasping for breath, extremely anxious, and yelling for help. Breath sounds are diminished in the right chest. The most appropriate first step is to: A. perform tracheal intubation B. Insert an oropharyngeal airway 6 | P a g e C. Perform needle decompression of the right chest D. Manually displace the gravid uterus to the left side of the abdomen E. Initiate 2 large-caliber peripheral IV lines and crystalloid infusion C. Perform needle decompression of the right chest Which one of the following findings in an adult should prompt immediate management during primary survey? A. Distended abdomen B. GCS of 11 C. Temperature of 36,5 D. Heart rate of 120 beats per minute E. Respiratory rate of 40 breaths per minute E. Respiratory rate of 40 breaths per minute A trauma patient present to your emergency department with inspiratory stridor and a suspected C- spine injury. Oxygen saturation is 88% on high-flow oxygen via a nonrebreathing mask. The most appropriate next step is to: A. Apply cervical traction B. Perform immediate tracheostomy C. Insert bilateral thoracostomy tubes D. Maintain 100% oxygen and obtain immediate c-spine x-rays E. Maintain inline immobilization and establish a definitive airway E. Maintain inline immobilization and establish a definitive airway When apply the Rule of Nines to infants, A. It is not reliable B. The body is proportionally larger in infants than in adults C. The head is proportionally larger in infants than in adults D. The legs are proportionally larger in infants than in adults E. The arms are proportionally larger in infants than in adults C. The head is proportionally larger in infants than in adults A 60-year-old man sustains a stab wound to the right posterior flank. Witnesses state the weapon was a small knife. His heart rate is 90 beats per minute, blood pressure is 128/72mmHg and respiratory rate is 24 breaths per minute. The most appropriate action to take at this time is to: A. Perform a colonoscopy B. Perform a barium enema C. Perform an intravenous pyelogram D. Perform serial physical examinations E. Suture repair the wound and outpatient follow up D. Perform serial physical examinations The following are criteria for transfer to a burn center, EXCEPT for: A. Partial-thickness and full-thickness burns on greater than 10% of the BSA 7 | P a g e B. Any full-thickness burn C. Partial-thickness and full-thickness burn involving the face, hands, feet, genitalia, perineum and skin overlying major joints D. Elevated central venous pressure E. Inhalation injury D. Elevated central venous pressure Systolic blood pressure starts to decrease in which class of hemorrhage? A. Class 0 B. Class 1 C. Class 2 D. Class 3 E. Class 4 C. Class 2 A 7-year-old boy is brought to the ED by his parents several minutes after he fell through a window. He is bleeding profusely from a 6-cm wound of his medial right thigh. Immediate management of the wound should consist of: A. Application of a torniquet B. Direct pressure on the wound C. Packing the wound with gauze D. Direct pressure on the femoral artery at the groin E. Debridement of devitalized tissue B. Direct pressure on the wound For the patient with severe traumatic brain injury, profound hypocarbia should be avoided to prevent: A. Respiratory alkalosis B. Metabolic acidosis C. Cerebral vasoconstriction with diminished perfusion D. Neurogenic pulmonary edema E. Shift of the oxyhemoglobin dissociation curve. C. Cerebral vasoconstriction with diminished perfusion A 33-year-old woman is involved in a head-on motor vehicle crash. It took 30 minutes to extricate her from the car. Upon arrival in the ED, her heart rate is 120 beats per minute, BP is 90/70mmHg, respiratory rate is 16 breaths per minute, and GCS is 15. Examination reveals bilaterally equal breath sounds, anterior chest wall ecchymosis, and distended neck veins. Her abdomen is flat, soft, and not tender. Her pelvis is stable. Palpable distal pulses are found in all 4 extremities. Of the following, the most likely diagnosis is: A. hemorrhagic shock B. Cardiac tamponade C. Massive hemothorax D. Tension pneumothorax E. Diaphragmatic rupture. 10 | P a g e B. Perform an arch aortogram C. Insert a second left chest tube D. Prepare for an exploratory thoracotomy E. Perform a chest CT D. Prepare for an exploratory thoracotomy A 56-year-old man is thrown violently against the steering wheel of his truck during a motor vehicle crash. On arrival in the ED he is diaphoretic and complaining of chest pain. His BP is 60/40mmHg and his respiratory rate is 40 breaths per minute. Which of the following best differentiates cardiac tamponade from tension pneumothorax as the cause of his hypotension? A. Tachycardia B. Pulse volume C. Breath sounds D. Pulse pressure E. Jugular venous pressure C. Breath sounds All of the following are true of the Mallampati classification EXCEPT: A. Class IV is the easiest intubation, while Class 1 is the most difficult B. It helps assess for difficult intubations C. It is part of the LEMON assessment D. It comprises a visual assessment of the distance from the tongue base to the roof of the mouth, and therefore the amount of space there is to work E. A poor Mallampati score is associated with a higher incidens of obstructive sleep apnea. A. Class IV is the easiest intubation, while Class 1 is the most difficult A 23-year-old man sustains three stab wounds to the upper right chest during an altercation and is brought by ambulance to a hospital that has full surgical capabilities. His wounds are all above the nipple. He is endotracheally intubates, closed tube thoracostomy is performed, and 2 liters of crystalloid solution are infused through 2 large-caliber IVs. His BP i 60/0mmHg, heart rate is 160 beats per minute, and respiratory rate is 14 breaths per minute (ventilated with 100% O2). 1500ml of blood has drained from the right chest. The most appropriate next step in managing this patient is to: A. Perform FAST B. Obtain a CT of the chest C. Perform angiography D. Urgently transfer the patient to the operating room E. Immediately transfer the patient to a trauma center. D. Urgently transfer the patient to the operating room Which of the following signs is LEAST reliable for diagnosing esophageal intubation? A. Symmetrical chest movement B. End-tidal CO2 presence by colorimetry C. Bilateral breath sounds 11 | P a g e D. Oxygen saturation > 92% E. ETT above carina on chest x-ray E. ETT above carina on chest x-ray Which one of the following signs necessitates a definitive airway in severe trauma patients? A. Facial lacerations B. Repeated vomiting C. Severe maxillofacial fractures D. Sternal fracture E. GCS score of 12 C. Severe maxillofacial fractures Which one of the following statements is correct? A. Cerebral contusion may coalesce to form an intracerebral hematoma B. Epidural hematomas are usually seen in frontal region C. Subdural hematomas are caused by injury to the middle meningeal artery D. Subdural hematomas typically have a lenticular shape on CTscan E. The associated brain damage is more severe in epidural hematomas. A. Cerebral contusion may coalesce to form an intracerebral hematoma An 18 year old male is brought to the ED after having been shot. He has one bullet wound just below the right clavicle and another just below the costal margin in the right posterior axillary line. His BP is 110/60, HR is 90bpm, and RR is 34bpm. After ensuring a patent airway and inserting 2 large caliber iv line, the next appropriate step is to: A. Obtain a chest x-ray B. Adminster a bolus of additional iv fluid C. Perform a laparotomy D. Obtain abdominal CT-scan E. Perform DPL A. Obtain a chest x-ray An 8 year old boy falls 4,5meters from a tree and is brought to the ED by his familiy. His vital signs are normal, but he complains of left upper quadrant pain. An abdominal CT scan reveals a moderately severe laceration of the spleen. The receiving institution does not have 24 hour a day operating room capabilities. The most appropriate management of this patient would be: A. Type and crossmatch for blood B. Request consultation of a pediatrician C. Transfer the patient to a trauma center D. Admit the patient to the ICU E. Prepare the patient for surgery the next day D. Admit the patient to the ICU A construction worker falls from a scaffold and is transferred to the ED. His HR is 124 bpm and BP is 85/60mmHg. He complains of lower abdominal pain. After assessing the airway and chest, immobilizing 12 | P a g e the c-spine and initiating fluid resuscitation, the next step is to perform A. FAST B. Detailed neurological exam C. Rectal exam D. Cervical c-spine x-ray E. Urethral catheterization. A. FAST A 22 year old male sustains a shotgun wound to the left shoulder and chest at close range. His BP is 80/40mmHg and his HR is 130bpm. After 2 liters of crystalloid solution are rapidly infused, his BP increases to 122/84, and HR decreases to 100bpm. He is tachypneic with RR of 28. On physical examination, his breath sounds are decreased at the left upper chest with dullness on percussion. A large caliber (36 french) tube thoracostomy is inserted in the fifth intercostal space with the return of 200ml of blood and no air leak. The most appropriate next step is to: A. insert a foley catheter B. Begin to transfuse o-negative blood C. Perform thoracotomy D. Obtain a CT-scan of chest and abdomen E. Repeat the physical examination of the chest E. Repeat the physical examination of the chest Which one of the following statements concerning spine and spinal cord trauma is true? A. A normal lateral c-spine film excludes injury B. A vertebral injury is unlikely in the absence of physical findings of a cord injury C. A patient with a suspected injury requires immobilization on a short spine D. Diaphragmatic breathing in an unconscious patient who has fallen is a sign of spine injury E. Determination of whether a spinal cord lesion is complete or incomplete must be made in the primary survey D. Diaphragmatic breathing in an unconscious patient who has fallen is a sign of spine injury A 20 year old athlete is involved in a motorcycle crash. When he arrives in the ED, he shouts that he cannot move his legs. On physical examination, there are noe abnormalities of the chest, abdomen or pelvis. The patient has no sensation in his legs and cannot move them, but his arms are moving. The patients RR is 28 bpm, HR is 88bpm and BP is 80/60mmHg. He is pale and sweaty. What is the most likely cause of this condition? A. Neurogenic shock B. Cardiogenic shock C. Abdominal hemorrhage D. Myocardial contusion E. Hyperthermia. A. Neurogenic shock A 28 year old male is brought to the ED. He was involved in a fight in which he was beaten with a wooden stick. His chest shows multiple severe bruises. His airway is clear, RR is 22, HR is 126 and systolic 15 | P a g e D. Widened symphysis pubis E. D. Widened symphysis pubis A 34-year-old man is brought to the ED after being pinned to the wall of building by a cement truck. He is in obvious shock, and has deformities and marked swelling of both thighs. Although no open wound are present, his shock: A. Cannot be explained without concomitant pelvic fracture B. Signifies a loss of approximately 15% C. Is consistent with blood loss from bilateral femoral fracture D. Will likely be reversed if appropriate traction splint are applied E. Cannot be explained by his observed injuries unless a major arterial injury exist C. Is consistent with blood loss from bilateral femoral fracture Prior to passage of urinary catheter in a man, it is essential to: A. Examine the abdomen B. Determine pelvic stability C. Examine the rectum and perineum D. Perform a retrograde urethrogram E. Know the history and mechanism of injury C. Examine the rectum and perineum The best guide for adequate fluid resuscitation of the burn patient is: A. Adequate urinary output B. Reversal of systemic acidosis C. Normalization of the heart rate D. A normal central venous pressure E. 4ml/kg/percent body burn/24 hours A. Adequate urinary output A 36-year-old woman is beaten about the head and face and is brought to the local community hospital in full spinal immobilization. Her BP is 13088, HR 70/minutes, and RR 18/minute. Pulse oximetry indicated 98% while she was given 100% O2 via a non rebreather mask. Her airway is clear. She has marked swellings on her face and several lacerations of her scalp that are not actively bleeding. She does not respond to verbal stimuli, but localizes to painful stiumuli and opens her eyes. She moves all extremities equally. The remainder of her physical exam is normal. There is no neurosurgeon at the local hospital. After ensuring the patient airway, the most appropriate course of action is to: A. Admit the patient to the hospital for observation B. Obtain x-ray of her facial bones prior to transfer C. Obtain complete x-ray evaluation of the cervical spine D. Transfer the patient to a neurosurgeon without performing a CT-scan E. Perform DPL or request abdominal ultrasonography D. Transfer the patient to a neurosurgeon without performing a CT-scan 16 | P a g e For the trauma patient with cerebral edema, hypercarbia should be avoided to prevent: A. metabolic acidosis B. Respiratory acidosis C. Cerebral vasodilatation D. Neurogenic pulmonary edema E. Reciprocal high level of PaCO2 C. Cerebral vasodilatation A 29 y/o male is brought to the ED after being involved in a motor vehicular collision when his car struck a bridge abutment. He is intoxicated, has GCS 13 and complains of abdominal pain. His BP was 80mmHg systolic by palpation on admission, but rapidly increased to 110/70 with the administration of IV fluid. His heart rate is 120/minute. The chest x-ray show loss of aortic know, widening of mediastinum, no rib fracture and no hemopneumothorax. Contrast angiography: A. Is not indicated B. Should be performed after CT scan of the chest C. Is positive ofr aortic rupture in 80% of similar cases D. Is not necessary if the CT-scan of the chest is normal E. Should be performed after DPL D. Is not necessary if the CT-scan of the chest is normal Important screening x-rays to obtain in the multiple system trauma patient are: A. Skull, chest and abdomen B. Chest, abdomen and pelvis C. Skull cervical spine and pelvis D. Cervical spine, chest and pelvis E. Cervical spine, chest and abdomen D. Cervical spine, chest and pelvis All of the following statement regarding pulse oxymetry are true EXCEPT A. excessive surrounding room light can interfere with the accuracy of the reading B. Significant levels of dysfunctional hemoglobin can affect the accuracy of the reading C. It provides a continuous measurement of the partial pressure of oxygen D. It is dependent on differential light absorption by oxygenated and deoxygenated hemoglobin E. It provides a continuous, non-invasiv measurement of pulse rate that is updated with each HR C. It provides a continuous measurement of the partial pressure of oxygen Bronchial intubation at the right or left mainstem bronchus can easily occuring during infant endotracheal intubation because A. The trachea is relatively short B. The distance from the lips to the larynx is relatively short C. The use of tubes without cuffs allow the tube to slip easily D. The mainstem bronchi are less angulated in their relation to the trachea E. Do litte friction exist between endotracheal tube and the wall of the trachea. 17 | P a g e A. The trachea is relatively short A 52 y/o woman sustaining 50% total body surface burns in an explosion. She has burns around the chest and both upper arms. Adequate resuscitation is initiated. She is nasotracheally intubated and is being mechanically ventilated. Her CarboxyHb level is 10%. Her arterial blood gas reveals PaO2 of 40mmHg, PaCo2 of 60mmHg and pH of 7,25. Appropriate immediate management at the time is to A. Ensure adequate tissue perfusion B. Increase the rate of fluid resuscitation C. Add PEEP D. Reassess for the presence of pneumothorax E. Administer IV narcotics in small amounts ?A. Ensure adequate tissue perfusion All of the following suggest urethral injury EXCEPT A. scrotal hematoma B. Blood in rectal lumen C. Blod in external urethral meatus D. High riding prostate on rectal exam E. Absence of a palpable prostate on rectal exam E. Absence of a palpable prostate on rectal exam Which one of the following is recommended method for threating frostbite? A. Moist heat B. Early amputation C. Padding and elevation D. Vasodilators and heparin E. Topical application of silversulphadiazine A. Moist heat A 32-year-old mans right leg is trapped beneath his overturned car for nearly two hours before he is extricated. On arrival in the ED, both lower extremities are cool, mottled, insensate and motionless. Despite normal vital signs, pulses cannot be palpated below the femoral vessels and the muscles of the lower extremities are firm and hard. During the initial management of this patient, which of the followin is most likely to improve chances for limb salvage? A. Apply skeletal traction B. Administering anticoagulant drugs C. Administering trombolytic thearpy D. Performing lower extremity fasciotomies E. Immediately transfer the patient to a trauma care D. Performing lower extremity fasciotomies A 26 y/o seat belted driver is brought to the ED after a car crash. Primary survey reveals no evidence of serious injury except for diffuse, mild abdominal tenderness. Bowel sounds are hypoactive and liver dullness is questionable. Abdominal films reveal free air. The patient should 20 | P a g e B. Oculovestibular reflex C. Gag reflex D. Argyll Robertson pupil What is the simplest way to open the airway in an unconscious patient? A. Pull out the tongue B. Tilt head and lift chin C. Lift neck from behind D. Jaw thrust What is the approximate time the brain can be anoxic before developing irreversible damage? A. 10 min B. 5 min C. 2 min D. 20 min You are treating a truama patient and attempt a definitive airway by intubation. However, the vocal cords are not visible. What tool would be the most valuable for achieving successful intubation? A. Gum elastic bougie B. Lateral cervical spine x-ray C. Nasopharyngeal airway D. Oxygen E. Laryngeal mask airway A. Gum elastic bougie A 79 year old female is involved in a motor vehicle crash and presents to the ED. She is on Coumadin and a beta blocker. Which of the following statements is true concerning her management? A. The risk of subdural hemorrhage is decreased B. Absence of tachycardia indicates that the patient is hemodynamically normal C. Non-operative management of abdominal injuries is more likely to be successful in older adults than in younger patients D. Vigorous fluid resuscitation may be associated with cardiorespiratory failure D. epinephrine should be infused immediately for hypotension D. Vigorous fluid resuscitation may be associated with cardiorespiratory failure The most common acid base disturbance encountered in injured pediatric patients is caused by: A. Hemorrhage B. Changes in ventilation C. Renal failure D. Injudicious bicarbonate administration E. Insufficient sodium chloride administration B. Changes in ventilation? A 17 year old female is brought to the ED following a 2 meter fall onto concrete. She is unresponsive and found to have a RR of 32, BP 90/60 and HR 68. The first step in treatment is: 21 | P a g e A. Adminstering vasopressors B. Establishing IV access for drug-assisted intubation C. Seeking the cause of her decreased level of consciousness D. Applying oxygen and maintaining airway E. Excluding hemorrhage as a cause of shock D. Applying oxygen and maintaining airway A 25 year old male is brought to the ED following a bar fight. He has altered consciousness, open his eyes on command, moans without forming discernible words, and localizes to painful stimuli. Which one of the following statements concerning this patient is true? A. Hyperoxia should be avoided B. CT scanning is an important part of neurological assessment C. Mandatory intubation to protect his airway is required D. His GCS suggest severe head injury E. His level of consciousness can be solely attributed to elevated blood alcohol. B. CT scanning is an important part of neurological assessment Han har GCS på 10-11, således passer det ikke med svaralternativ D som tilsvarer GCS <9. Ettersom GCS er >8 passer heller ikke svaralternativ C. E er feil da det ikke kan gi så lav GCS Which one of the following statements regarding genitourinary injuries is true? A. Urethral injuries are associated with pelvic fractures B. All patients with microscopic hematuria require evaluation of genitourinary tract C. Patient presenting with gross hematuria and shock will have a major renal injury as the source of hemorrhage D. Intraperitoneal bladder injuries are usually managed definitively with a urinary catether E. Urinary catheters should be placed in all patients with pelvic fractures during the primary survey A. Urethral injuries are associated with pelvic fractures B er feil - det er pasienter med makroskopisk hematuri og/eller pas. med mikroskopisk hematuri og sjokk som det er aktuelt å gjøre CT av. C kan være riktig, men kan ikke si det sikkert at det er nyrene som er blødningskilden. D er feil - ekstraperitoneal blæreskade behandles med kateter, og E er feil da man først må undersøke for uretraskade Cardiac tamponade: A. Requires surgical intervention B. Is defintively managed by needle pericardiocentesis C. Is easily diagnosed by discovery of Becks triad in the ED D. Is indicated by Kussmaul breathing E. Is most common with blunt thoracic trauma and anterior rib fractures A. Requires surgical intervention A 6 month old infant, being held in her mothers arms, is ejected on impact from a vehicle that is struck head on by an oncoming car traveling at 65kph. The infant arrives in the ED with multiple facial injuries, is lethargic, and is in severe respiratoy distress. Respiratory support is not effective using a bag mask 22 | P a g e device, and her oxygen saturation is falling. Repeated attempts at orotracheal intubation are unsuccessful. the most appropriate procedure to perform next is: A. perform needle cricothyroidotomy with jet insufflation B. Administer heliox and racemic epinephrine C. Perform nasotracheal intubation D. Perform surgical cricothyroidotomy E. Repeat orotracheal intubation A. perform needle cricothyroidotomy with jet insufflation Which one of the following injuries is adressed in the secondary survey? A. Bilateral femur fractures with obvious deformity B. Open fracture with bleeding C. Mid thigh amputation D. Unstable pelvic fracture E. Forearm fracture E. Forearm fracture A 22 year old male present following a motorcycle crash. He complains of the inability to move his legs. His BP is 80/50, HR 70, RR 18 and GCS 15. Oxygen saturation is 99% on 21 nasal prongs. Chest x-ray, pelvic x-ray and FAST are normal. Extremities are normal. His management should be: A: 2L of iv . crystalloid and two units of pRBCs B. 2L of iv crystalloid and vasopressors if BP does not respond C. 2L of iv. crystalloid, mannitol and iv steroids D. Vasopressors and laparotomy E. 1 unit of albumin and compression stockings B. 2L of iv crystalloid and vasopressors if BP does not respond A 35 year old female sustains multiple injuries in a motor vehicle crash and is transported to a small hospital in full spinal protection. She has a GCS of 4 and is being mechanically ventilaed. I.v access is established and warmed crystalloid is infused. She remains hemodynamically normal and full spinal protection is maintained. Preparations are made to transfer her to another facility for definitive neurosurgical care. Prior to transport, which of the following tents or treatments is mandatory? A. FAST exam B. Lateral cervical spine xray C. Chest x-ray D. Administration of methylprednisolon E. CT of abdomen C. Chest x-ray A 22 year old male is assaulted in a bar. A semi-rigid cervical collar is applied and he is immobilized on a spine board. On initial examination, his vital signs are normal, and his GCS is 15. Which of the following is an indication for CT in this patient with possible minor traumatic brain injury? A. Blood alcohol concentration of 0,16% B. Presence of an isolated 10cm scalp laceration 25 | P a g e A. Fetal assessment should take priority B. Log-rolling the patient to the right will decompress the vena cava C. Rh-immunoglobulin therapy should be immediately administered D. Vasopressors should be given to the patient E. The patient has likely impending respiratory failure. E. The patient has likely impending respiratory failure. Minute ventilation increases primarily due to an increase in tidal volume. Hypocapnia (PaCO2 of 30 mm Hg) is therefore common in late pregnancy. A PaCO2 of 35 to 40 mm Hg may indicate impending respiratory failure during pregnancy. Lateral cervical spine film: A. Must be performed in the primary survey B. Can exclude any significant spinal injury C. Are indicated in all trauma patients D. Should be combined with clinical exam, AP and odontoid, or CT E. Require the following films: oblique views, AP, odontoid and flexion extention views prior to spinal clearance in trauma patients D. Should be combined with clinical exam, AP and odontoid, or CT A 30 year old male is stabbed in the right chest. On arrival in the ED, he is very short of breath. His HR is 120bpm, BP is 80/50. His neck veins are flat. On auscultation of the chest, there is diminshed air entry on the right side, and there is dullness posteriorly on percussion. These findings are most consistent with: A. Hemothorax B. pericardial tamponade C. Tension pneumothorax D. Hypovolemia from the liver injury E. Spinal cord injury A. Hemothorax A specific aspect of the treatment of thermal injury is: A. Chemical burn require the immediate removal of clothing B. Patients who sustain thermal injury are at lower risk for hypothermia C. Patients with circumferential truncal burns need prompt fasciotomies D. Electrical burn are associated with extensive skin necrosis (from entry point to exit) E. The parkland formula should be used to determine adequacy of resucitation. A. Chemical burn require the immediate removal of clothing A 15 year old male is brought to the ED after being involved in a motor vehicle crash. He is unconscious and was intubated at the scene by emergency personnel. Upon arrival at the ED, the patients oxygen saturation is 92%, HR is 96 bpm and BP is 150/85 Breath sounds are decreased on the left side of the thorax. The next step is: A. Immediate needle crycothyroidotomy B. Reassess the position of the endotracheal tube 26 | P a g e C. Chest tube insertion D. Immediate needle thoracentesis E. Obtain a chest x-ray B. Reassess the position of the endotracheal tube The first priory in management of a long bone fracture is: A. Reduction of pain B. Prevention of infection in case of an open fracture C. Prevention of further soft tissue injury D. Improve long term function E. Control of hemorrhage E. Control of hemorrhage A 30 year old female is brought to the ED after being injured in a motor vehicle crash. Her initialt BP is 90/60mmHg, and her HR is 122bpm. She responds to rapid infusion of 1L crystalloid solution with a rise in her BP to 118/88 and decrease in her HR to 90bpm. Her pressure suddenly decreased to 96/66. The least likely cause of her hemodynamic change is: A. Ongoing blood loss B. Blunt cardiac injury C. Traumatic brain injury D. Inadequate resuscitation. E. Tension pneumothorax C. Traumatic brain injury Limb-threatening extremity injuries: A. Require a torniquet B. Should be defintively managed by application of a traction split. C. Are rarely present without an open wound D. Are characterized by the presence of ischemic or crushed tissue. E. Indicate a different order or priorities for the patients initial assessment and resuscitation. B. Should be defintively managed by application of a traction split. A 29 year old female arrives in the ED after being involved in a motor vehicle crash. She is 30 weeks pregnant. She was restrained with a lap and shoulder belt, and an airbag deployed. Which one of the following statement best decribes the risk of injury? A. Deployment of the airbag increased the risk of fetal loss B. The risk of premature fetal delivery and death is reduced by the use of restraints C. The use of seatbelts is associated with increased risk of maternal death. D. The mechanism of injury suggest the need for emergency ceasarean section due to the risk of impending abruptio placentae E. The deployment of the airbag increases the risk of maternal abdominal injury B. The risk of premature fetal delivery and death is reduced by the use of restraints 27 | P a g e Supraglottic airway devices: A. Are equivalent to endotracheal intubation B. Require neck extension for proper placement C. Are preferable to endotracheal intubation in a patient who cannot lie flat D. Are of value as part of a difficult or failed intubation plan E. Provide one form of definitive airway D. Are of value as part of a difficult or failed intubation plan A 25 year old male is brought to the hospital after sustaining partial and full thickness burns involving 60% of his body surface area. His right arm and hand are severely burned. There are obvious full thickness burns of the entire right hand and a circumferential burn of the right arm. Pulses are absent at the right right wrist and are not detected by doppler examination. The first step in management of the right upper extremity should be: A. Fasciotomy B. Angiography C. Escharatomy D. Heparinization E. Tangential excision C. Escharatomy Hypertension following a headinjury: A. Should be treated to reduce intracranial pressure B: Indicates pre-existing hypertension C. May indicate imminent herniation from critically high intracranial pressure D. Mandates prompt administration of mannitol E. Should prompt burr hole drainage of potentialt subdural hematomas. C. May indicate imminent herniation from critically high intracranial pressure Initial treatment of frostbite injuries involves: A. application of dry heat B. Debridement of hemorrhagic blisters C. Early amputation to prevent septic complications D Rapid rewarming of the body part in circulation warm water E. Massage of the affected area D Rapid rewarming of the body part in circulation warm water Signs and symptoms of airway compromise include all of the following, EXCEPT: A. change in voice B. Stridor C. Tachypnea D. Dyspnea and agitation E. Decreased pulse pressure E. Decreased pulse pressure 30 | P a g e distended. Breath sounds are normal. Heart sounds are diminshed, iv access has been established and warm crystalloid is infusing. The next most important aspect of immediate management is: A. CT scan of the chest B. 12 lead ECG C. Left tube thoracostomy D. Begin infusion of RBCs E. FAST exam E. FAST exam For å se etter tamponade? Normale resp.lyder bilateralt taler mot trykkpneumothorax. EKG vil ikke gi noe informasjon. Thoracostomy ikke indisert da det ikke er mistenkt pneumo/hemothorax. CT uaktuelt pga hemodynamisk ustabil A 47 year old house painter is brought to the hospital after falling 6 meters from a ladder and landing straddled on a fence. Examination of his perineum reveals extensive ecchymosis. There is blood in the external urethreal meatus. The initial diagnostic study for evaluation of the urinary tract in this patient should be: A. Cystoscopy B. Cystography C. IV pyelography D. CT scan E. Retrograde urethrography E. Retrograde urethrography Neurogenic shock has all of the following classic characteristics except which one: A. Hypotension B. Vasodilatation C. Bradycardia D. Neurologic deficit E. Narrowed pulse pressure E. Narrowed pulse pressure Which one of the following should be performed first in any patient whose injuries may include multiple closed extremity fractures? A. A thorough assessment of four limb perfusion B. Maneuvers to prevent necrosis of the skin C. Extremity compartment syndrome release D. Ensuring adequate oxygenation and ventilation E. Evaluation for occult crush syndrome D. Ensuring adequate oxygenation and ventilation Which of the following is the recommended Method for trestemt frostbite? A. Vasodilators B. Anticigulants C. Warm (40 degrees) water 31 | P a g e D. Padding and elevation E. Application of heat from a hairdryer C. Warm (40 degrees) water Which of the following physical findings suggest a cause of hypotension other than spinal cord injury? A. Prispism B. Bradycardia C. Diaphragmatic breathing D. Presence of deep tendon reflexes E. Ability to flex forearms but not extend them D. Presence of deep tendon reflexes. Spinal shock refers to loss of muscle toe (flaccidty) and loss of reflexes. The primary indication for transferring A patient to a higher level trauma center is: A. Unavailibility of surgeon or operating staff B. Multiple system injuries, including severe head injury C. Resource limitations as determined by the transferring doctor D. Resource limitations as determined by the hospital administration E. Widened mediastinum on chest x-ray following blunt trauma C. Resource limitations as determined by the transferring doctor (MÅ SJEKKES) A young man sustains a rifle wound to the mid-abdomen. He is brought promptly to the ED by prehospital personnel. His skin is cool and diaphoretic, and his systolic blood pressure is 58mmHg. Warmed crystalloid fluids are initiated without improvement in his vital signs. The next, most appropriate, step is to perform: A. a laparotomy B. An abdominal CT-scan C. Diagnostic laparoscopy D. Abdominal ultrasonography E. A diagnostic peritoneal lavage A. Laparotomy because of hemodynamic abnormality A 42-year-old man is trapped from the waist down beneath his overturned tractor for several hours before medical assistance arrives. He is awake and alert until just before arriving in the ED. He is now unconscious and responds only to painful stimuli by moaning. His pupils are 3mm in diameter and symmetrically reactive to light. Prehospital personnel indicate that they have not seen the patient move either of his lower extremities. On examination in the ED, no movement of his lower extremities are detected, even in response to painful stimuli. The most likely cause for this finding is: A. An epidural hematoma B. A pelvic fracture C. Central cord syndrome D. Intracerebral hemorrhage E. Bilateral compartment syndrome 32 | P a g e MÅ SJEKKES A 6-year-o boy is struck by an automobile and brought to the ED. He is lethargic, but withdraws purposefully from painful stimuli. His blood pressure is 90mmHg systolic, heart rate 140 beats per minute and his respiratory rate is 36 breaths per minute. The preferred route of venous access in this patient is: A. Percutaneous femoral vein cannulation B. Cutdown on the saphenous vein at the ankle C. Intraosseous catheter placement in the proximal tibia D. Percutaneous peripheral veins in the upper extremities E. Central venous access via the subclavian or internal jugular vein D. Percutaneous peripheral veins in the upper extremities A young man sustains a gunshot wound to the abdomen and is brought promptly to the ED by prehospital personnel. His skin is cool and diaphoretic, and he is confused. His pulse is thready and his femoral pulse is only weakly palpable. The definitive treatment in managing this patient is to: A. Administer O-negative blood B. Apply external warming devices C. Control internal hemorrhage operatively D. Apply a pneumatic antishock garment (PASG) E. Infuse large volumes of intravenous crystalloid solutions. C. Control internal hemorrhage operatively Regarding shock in the child, which of the following is FALSE? A. Vital signs are age-related B. Children have greater physiologic reserves than do adults C. Tachycardia is the primary physiologic response to hypovolemia D. The absolute volume of blood loss required to produce shock is the same as in adults E. An initial fluid bolus for resuscitation should approximate 20ml/kg Ringers Lactate D. The absolute volume of blood loss required to produce shock is the same as in adults A 33-year-old man is struck by a car travelling at 56km/h (35mph). He has obvious fractures of the left tibia near the knee, pain in the pelvic area, and severe dyspnea. His heart rate is 182 beats per minute, and his respiratory rate is 48 breaths per minute with no breath sounds heard in the left chest. A tension pneumothorax is relieved by immediate needle decompression and tube thoracostomy. Subsequently, his heart rate decreases to 144 beats per minute, his respirartory rate decreases to 36 breaths per minute and his blood pressure is 81/53 mmHg. Warmed Ringers lactate is adminstered intravenously. The next priority should be to: A. Perform external fixation of the pelvis B. Obtain abdominal and pelvic CT-scans C. Perform arterial embolization of the pelvic vessel D. Perform diagnostic peritoneal lavage or FAST E. Perform a urethrogram and cystogram D. Perform diagnostic peritoneal lavage or FAST 35 | P a g e A. Left hemothorax A 23-year-old man is brought immediately to the ED from the hospitals parking lot where he was shot in the lower abdomen. Examination reveals a single bullet wound. He is breathing and has a thready pulse. However, he is unconsious and has no detectable blood pressure. Optimale immediate management is to: A. Perform a diagnostic peritoneal lavage B. Initiate infusion of packed red blood cells C. Insert a nasogastric tube and urinary catheter D. Transfer the patient to the operating room, while initiating fluid therapy E. Initiate fluid therapy to return his blood pressure to normotensive D. Transfer the patient to the operating room, while initiating fluid therapy A teen-aged bicycle rider is hit by a truck traveling at high speed. In the ED, she is actively bleeding from open fractures of her legs, and has abrasions on her chest and abdominal wall. Her blood pressure is 80/50 mmHg, heart rate is 140 beats per minute, respiratory rate is 8 breaths per minute, and GCS score is 6. The first step in managing this patient is to: A. Obtain a lateral cervical spine x-ray B. Insert av central venous pressure line C. Adminster 2 liters of crystalloid solution D. Perform endotracheal intubation and ventilation E. Apply a pneumatic antishock garment (PASG) and inflate the leg compartments. D. Perform endotracheal intubation and ventilation An 8-year-old boy falls 4,5 meters (15 feet) from a tree and is brought to the ED by his family. His vital signs are normal, but he complains of left upper quadrant pain. An abdominal CT-scan reveals a moderately severe laceration of the spleen. The receiving institution does not have 24-hour-a-day operating room capabilities. The most appropriate management of this patient would be to A. Type and crossmatch for blood B. Request consultation of a pediatrician C. Transfer the patient to a trauma center D. Admit the patient to the intensive care unit E. Prepare the patient for surgery the next day D. Admit the patient to the intensive care unit Which of the following statements regarding injury to the central nervous system in children is TRUE? A. Children suffer spinal cord injury without x-ray abnormality more commonly than adults. B. An infant with a traumatic brain injury may become hypotensive from cerebral edema C. Initial therapy for the child with traumatic brain injury includes the administration of methylprednisolone intravenously D. Children have more focal mass lesions as a result for traumatic brain injury when compared to adults. E. Young children are less tolerant of expanding intracranial mass lesions than adults A. Children suffer spinal cord injury without x-ray abnormality more commonly than adults. 36 | P a g e A 17-year-old helmeted motorcyclist is struck broadside by an automobile at an intersection. He is unconscious at the scene with a blood pressure of 140/90mmHg, heart rate of 90 beats per minute, and respiratory rate of 22 breaths per minute. His respirations are sonorous and deep. His GCS score is 6. Immobilization of the entire patient may include the use of all the following EXCEPT: A. Air splints B. Bolstering devices C. A long spine board D. A scoop-style stretcher E. A semi-rigid cervical collar A. Air splints Twenty-seven patients are seriously injured in an aircraft accident at a local airport. The basic principle of triage should be to: A. Treat the most severely injured patients first B. Establish a field triage area directed by a doctor C. Rapidly transport all patients to the nearest appropriate hospital D. Treat the greatest number of patients in the shortest period of time E. Produce the greatest number of survivors based on available resources E. Produce the greatest number of survivors based on available resources An electrician is eletrocuted by a downed power line after a thunderstorm. He apparently made contact with the wire at the level of the right mid thigh. In the ED, his vital signs are normal and no dysrythmia is noted on ECG. On examination, there is an exit wound on the bottom of the right foot. His urine is positive for blood by dipstick but not RBCs are seen microscopically. Initial management should include: A. Immediate angiography B. Aggressive fluid infusion C. Intravenous pyelography D. Debridement of necrotic muscle E. Admission to the ICU for observation B. Aggressive fluid infusion - suspected rhabdomyolyse A young woman sustains a severe head injury as the result of a motor vehicular crash. In the ED, her GCS is 6. Her blood pressure is 140/90 mmHg and her heart rate 80 beats per minute. She is intubated and is being mechanically ventilated. Her pupils are 3mm in size and equally reactive to light. There is no other apparent injury. The most important principle to follow in early management of her head injury is to: A. Administer an osmotic diuretic B. Prevent secondary brain injury C. Agressively treat systemic hypertension D. Reduce meatbolic requirements of the brain E. Distinguish between intracranial hematoma and cerebral edema. B. Prevent secondary brain injury To establish a diagnosis of shock, A. Systolic blood pressure must be below 90mmHg 37 | P a g e B. The presence of a closed head injury should be excluded C. Acidosis should be present by arterial blood gas analysis D. The patient must fail to respond to intravenous fluid infusion E. Clinical evidence of inadequate organ perfusion must be present. E. Clinical evidence of inadequate organ perfusion must be present. A 32-year-old is brought to the hospital unconscious with severe facial injuries and noisy respirations after an automobile collision. In the ED, he has no apparent injury to the anterior aspect of his neck. He suddenly becomes apneic, and attempted ventilation with a face mask is unsuccessful. Examination of his mouth reveals a large hematoma of the pharynx with loss of normal anatomic landmarks. Initial management of his airways should be consist of: A. Inserting an oropharyngeal airway B. Inserting a nasopharyngeal airway C. Performing a surgical cricothyroidotomy D. Performing fiberoptic-guided nasotracheal intubation E. Performin orotracheal intubation after obtaining a lateral c-spine x-ray A. Inserting an oropharyngeal airway A 25-year-old woman is brought to the ED after a motor vehicle crash. She was initially lucid at the scene and then developed a dilated pupil and contralateral extremity weakness. In the ED, she is unconscious and has a GCS score of 6. The initial management step for this patient should be to: A. Obtain a CT-scan of the head B. Administer decadron 20mg IV C. Perform endotracheal intubation D. Administer mannitol 1g/kg IV E. Perform an emergency bone flap craniotomy on the side of the dilated pupil. C. Perform endotracheal intubation A contraindication to nasogastric intubation is the presence of a: A. Gastric perforation B. Diaphragmatic rupture C. Open depressed skull fracture D. Fracture of the cervical spine E. Fracture of the cribiform plate E. Fracture of the cribiform plate An 8-year-old girl is an unrestrained passenger in a vehicle struck from behind. In the ED, her blood pressure is 80/60mmHg, heart rate is 80 beats per minute, and respiratory rate is 16 breaths per minute. Her GCS score is 14. She complains that her legs feel "funny and wont move right". However, her spine x-rays do not show a fracture or dislocation. A spinal cord injury in this child: A. Is most likely a central cord syndrome B. Must be diagnosed by magnetic resonance imaging C. Can be excluded by obtaining a CT-scan of the entire spine 40 | P a g e D. Urinary output E. Jugular venous pressure D. Urinary out The displacement of tissue away from the path of a projectile, both temporarily and permanently, is known as: A. Conization B. Cavitation C. Crepitation D. Contusion B. Cavitation The single most important factor in determining the potential for injury due to energy exchange is: A. Mass of the bodies involved B. Velocity of the bodies involved C. Density of the tissues involved D. Surface area of the impact involved B. Velocity of the bodies involved In the management of shock, isotonic crystalloid solutions, such as Ringer's, are preferred because: A. The protein molecules in crystalloid solutions act as volume expanders B. These fluids draw interstitial fluid into the vascular space to enhance volume C. These solutions will stay in the vascular space longer than water solutions, such as D5W D. Their pH enhance oxygen delivery to the tissues C. These solutions will stay in the vascular space longer than water solutions, such as D5W With respect to the distance of a fall, which of the following is a guideline for determining a critical fall? A. 3 times the height of the patient B. 2 times the height of the patient C. 5 times the height of the patient D. 1 ½ times the height of the patient A. 3 times the height of the patient The phase of an explosion, or blast, in which hollow organs are squeezed and may rupture is called the __________ phase. A. Tertiary phase B. Quaternary phase C. Secondary phase D. Primary phase 41 | P a g e D. Primary phase During the primary survey and management of a trauma patient, the E in ABCDE stands for _________? A. Edema B. Eyes & ears C. Expose/Environment D. Electrical therapy C. Expose/Environment The time in which surgical intervention can make a difference in patient outcome is the __________? A. Golden period B. Golden time C. Golden era D. Golden minutes A. Golden period (hour) In the absence of extenuating circumstances, the maximum amount of time it should take to identify and manage immediate threats to life, prepare the patient for transport and begin transport is _________? A. 5 minutes B. 10 minutes C. 15 minutes D. 30 minutes B. 10 minutes In which of the following situations is the use of a short spinal immobilization device indicated? A. 28 year old male, unrestrained driver in a frontal impact crash. Awake, asks repeatedly what happened, complains of a headache, has a hematoma on his forehead. BP 122/84, HR 92, VR 20. B. 40 year old female who was pushed down a flight of stairs and is lying prone on the landing between two flights of stairs, complaining of back pain. BP 118/78, HR 100, VR 20. C. 17 year old female, restrained driver in a frontal impact crash. Awake, pale and diaphoretic, complains of upper right quadrant abdominal pain. BP 100/70, HR 108, VR 20. D. None of the above D. None of the above Your patient is a 32 year old man, restrained driver of a vehicle that has been involved in a frontal impact with a concrete bridge abutment. The patient is awake, but has difficulty answering questions due to shortness of breath. His ventilatory rate is 30 per minute. Of the following choices, when is the first time the patient's breath sounds should be checked? A. As soon as he is removed from the vehicle 42 | P a g e B. Prior to being removed from the vehicle C. As soon as he is immobilized on a long backboard D. Enroute to the trauma center B. Prior to being removed from the vehicle Which of the following represents adequate spontaneous ventilation in an adult? A. Tidal volume 100 mL, ventilatory rate 40/minute B. Tidal volume 500 mL, ventilatory rate 8/minute C. Tidal volume 300 mL, ventilatory rate 16/minute D. Tidal volume 600 mL, ventilatory rate 12/minute D. Tidal volume 600 mL, ventilatory rate 12/minute Which of the following is 100% accurate in verifying endotracheal tube placement? A. Pulse oximetry B. End-tidal capnometry C. Syringe aspiration D. None of the above D. None of the above....(added info by ang: to verify you need breath sounds, condensation in the tube and a confirmatory CXR) When utilizing percutaneous transtracheal ventilation, the correct ration of lung inflation to lung inflation time, in seconds, is: A. 1:4 B. 1:5 C. 1:2 D. 2:2 A. 1:4 Which of the following is a possible complication of using a manually triggered oxygen powered device for ventilation? A. Gastric distention B. Pneumothorax C. Inability to feel lung compliance D. All of the above D. All of the above 45 | P a g e C. Edema D. Reduced capillary blood flow A. Systemic acidosis A trauma patient who has fallen 20 feet from an apartment balcony is alert with warm, dry, pink skin, with normal capillary refilling time to the lower extremities, and is hypotensive. The upper extremities are cool, pale and diaphoretic. Which of the following injuries should be suspected? A. Aortic dissection B. Liver laceration C. Fractured pelvis D. Spinal cord injury D. Spinal cord injury Which of the following is a limitation of prehospital fluid resuscitation of the patient in hemorrhagic shock? A. Inability of fluids to carry oxygen B. Pulmonary edema C. Increased hemorrhage D. All of the above D. All of the above Which of the following statements regarding signs of intraabdominal injury is NOT true? A. Fresh blood in the abdominal cavity does not cause signs of peritonitis B. A significant amount of blood loss occurs before abdominal distention can be noticed C. Substantial intraabdominal hemorrhage always causes tenderness and abdominal rigidity D. Signs and symptoms of shock greater than can be explained by other injuries is a reliable indicator of intraabdominal injury C. Substantial intraabdominal hemorrhage always causes tenderness and abdominal rigidity Which of the following assessment techniques is least useful in the prehospital assessment of the patient with suspected intraabdominal trauma? A. Palpation B. Auscultation C. Inspection D. Scene assessment B. Auscultation 46 | P a g e Pregnant trauma patients should be placed on the left side because: A. This prevents seizures due to eclampsia B. This prevents abruption of the placenta C. This prevents compression of the vena cava D. This is the best way to auscultate fetal heart tones C. This prevents compression of the vena cava Survival of the fetus in a trauma situation is most dependent upon which of the following factors? A. Gestational age of the fetus B. Prenatal care C. Immediate cesarean section D. Good resuscitation of the mother D. Good resuscitation of the mother A patient struck in the back of the head with a baseball bat may sustain a cerebral contusion to which area of the brain? A. Frontal and occipital B. Occipital C. Parietal D. Frontal B. Occipital (probably too low velocity for Coup & Contre Coup) In the context of the caring for an adult patient with traumatic brain injury who is deteriorating and exhibiting signs of herniation, hyperventilation means ventilating with a BVM and 100% oxygen at a rate of: A. 12 to 16 per minute B. 32 to 40 per minute C. 8 to 12 per minute D. 20 per minute D. 20-hyperventilate to temp decrease ICP In the United State, which of the following mechanisms most frequently causes spinal cord injury in adults? A. Shallow water diving B. Vehicle crashes C. Falls D. Pedestrian struck by a vehicle 47 | P a g e B. Vehicle crashes Which of the following presentation indicate spinal cord injury? A. Complete loss of sensory and motor function below the site of injury B. Weakness and parethesia in the upper extremities, but normal function in the lower extremities C. Complete loss of function on one side of the body and loss of pain and temperature sensation on the opposite side D. All of the above D. All of the above Of the following, which is the earliest indication of compartment syndrome? A. Paralysis of the affected muscles B. Loss of pulses C. Loss of feeling in the web space between the thumb and index finger or between the first and second toes D. Tense swelling of the involved area D. Tense swelling of the involved area A traction splint may be used for which of the following injuries? A. Knee dislocation B. Pelvic fractures C. Femur fractures D. All of the above C. Femur fractures Which of the following descriptions meets the criteria for transport to a facility with a burn unit? A. A 49 year old female with a partial thickness burn from her elbow to her shoulder B. A 25 year old male with an electrical burn across his chest C. A 9 year old make with superficial burns on the backs of both legs D. A 32 year old female with a partial thickness burn about twice the size of her hand on her back B. A 25 year old male with an electrical burn across his chest The preferred method of dressing burns in the prehospital setting is: A. Dry sterile dressing B. Moist sterile dressing 50 | P a g e C. Patient complaining of pain on palpation of the neck D. Patient with a knife wound to the chest and a GCS of 15 D. Patient with a knife wound to the chest and a GCS of 15 The premise of PHTLS is that: A. EMTs must treat all trauma patient based on protocols B. EMTs are capable of sound patient care judgment, given an adequate knowledge base C. EMTs must work only under on-line medical direction when caring for trauma patients D. EMTs are capable of working independently of medical direction B. EMTs are capable of sound patient care judgment, given an adequate knowledge base Which of the following warrants classification of a patient as a critical trauma patient? A. An extremity fracture B. Preexisting major medical problem C. Bleeding from the nose D. Deployment of air bags in a motor vehicle crash B. Preexisting major medical problem Which of the following is the foundation of effective trauma care? A. Protocols permitting invasive airway procedures B. The ability to administer large amounts of crystalloid fluids C. The ability to quickly locate and manage life-threatening and potentially life-threatening injuries D. Effective spinal immobilization skills C. The ability to quickly locate and manage life-threatening and potentially life-threatening injuries You arrive at the scene of a motor vehicle collision in which a vehicle struck a tree. Which is the best indicator of potential injury? A. Circumference of the vehicle B. Diameter of the tree C. Mass of the vehicle D. Speed of the vehicle D. Speed of the vehicle The potential for death or serious injury is greatest in which of the following motor vehicle collisions? A) Down and under B) Ejection from vehicle 51 | P a g e C) Lateral compression D) Up and over B) Ejection from vehicle Bilateral femur fractures are most often associated with which type of motorcycle crash? A) Angular impact B) Bike-road impact C) Head-on impact D) Rear impact C) Head-on impact Which is the preferred fluid for resuscitation of hemorrhagic shock in the prehospital setting? A) 5% dextrose in water B) 7.5% hypertonic saline C) Hetastarch D) Lactated Ringer's D) Lactated Ringer's Which is the most common cause of upper airway obstruction in the trauma patient? A) Blood B) Teeth C) Tongue D) Vomitus C) Tongue Which is the preferred adjunct device for verifying placement of an endotracheal tube in a patient with a perfusing rhythm? A) End-tidal CO2 monitoring (capnography) B) Esophageal detector device C) Pulse oximeter D) Stethoscope A) End-tidal CO2 monitoring (capnography) Which is the most important reason to maintain an open airway in the trauma patient? A) Prevents aspiration and pneumonia B) Prevents hypoxemia and hypercarbia C) Prevents snoring respirations D) Prevents the tongue from blocking the pharynx 52 | P a g e B) Prevents hypoxemia and hypercarbia Essential airway skills include manual clearing of the airway, manual maneuvers, suctioning and which of the following? A) Dual lumen airway B) Endotracheal intubation C) Laryngeal mask airway D) Oropharyngeal airway D) Oropharyngeal airway Your patient is a middle aged male who crashed his motorcycle. He is unresponsive. After opening the airway using a modified jaw thrust, you note the patient has respirations at a rate of 6. Auscultation reveals breath sounds are absent on the left side. Which of the following is the most appropriate next intervention? A) Apply a non-rebreather mask B) Begin ventilation with a BVM C) Insert an endotracheal tube D) Perform a needle decompression B) Begin ventilation with a BVM Which best describes shock? A) Decreased Glasgow Coma Scale (GCS) B) Flushed, dry, hot skin combined with bradycardia C) Generalized inadequate tissue perfusion D) Low blood pressure combined with tachycardia C) Generalized inadequate tissue perfusion Your patient has a deep laceration to his antecubital fossa with significant bleeding. What is the most appropriate initial action? A) Apply a tourniquet B) Apply direct pressure C) Initiate rapid transport D) Restore blood volume B) Apply direct pressure Hypotension of unknown etiology in a trauma patient should be assumed to result from which of the following? A) Blood loss B) Cardiac tamponade 55 | P a g e The most immediate life threatening condition resulting from injury to solid abdominal organs is which of the following? A) Acute respiratory failure B) Hemorrhage. C) Multiple organ failure. D) Peritonitis B) Hemorrhage. An adult male sustained a deep laceration to his distal thigh. Bright red blood is spurting from the wound. Direct pressure is not controlling the bleeding. What is the most appropriate next step? A) Apply a topical hemostatic agent and transport B) Apply a tourniquet and tighten it until bleeding stops C) Elevate the leg and apply pressure to the femoral artery D) Maintain direct pressure and transport immediately B) Apply a tourniquet and tighten it until bleeding stops An 18-year-old female was struck by a car and has sustained an apparent left femur fracture. Communication with her is hampered because she only speaks a foreign language. Which finding, by itself, does not mandate immobilization of the cervical spine? A) Fracture of the femur B) Inability to communicate C) Mechanism of injury D) Tenderness over the cervical spine C) Mechanism of injury During the primary survey of a trauma patient, you note that the patient is agitated and confused, and has multiple injuries from an altercation. Which of the following choices is the most appropriate first treatment priority? A) Blood glucose determination B) Correction of possible hypoxia C) Full immobilization to a backboard D) Obtain intravenous access B) Correction of possible hypoxia When something crashes in secondary survey, you... GO BACK TO PRIAMRY SURVEY 56 | P a g e If the question says you do not have capabilities, you will most likely PREPARE FOR AND DO NOT DELAY TRANSPORT How much blood can femur lose 2L What is always going to be the shock type to pick Hemorrhagic What are the classes of hemorrhagic shock 0-15% is class 1 15-30 is class 2 30-40% is class 3 >40% is class 4 What is the EARLIEST predictor of shock at what class Narrowed pulse pressure in class 2 What will be base deficit for class 2 shock? -2 to -6 meQ What will be the deficit for class 4? -10 or less When will urine output start to decrease in shock? Class 3 and 4 When does BP start to fall in shock Class 3 What is GCS? E-4 V-5 M-6 Eye spont 4 To command 3 To pain 2 None 1 Verbal spont is 5 Verbal confused is 4 Inappropriate 3 57 | P a g e Garbling is 2 None is 1 Motor spont is 6 Localize pain is 5 Withdrawal pain is 4 Decorticating (flexion) is 3 Decebrate (extension) is 2 None is 1 Two main reasons to think about why GCS goes down? TBI Hemorrhagic shock Why do you want to put in chest tube before needle decompression? SIMPLE PTX can turn into TENSION PTX When is DPL useful over fast? No ultrasound available. What is LEMON for airway patency Look Evaluate Malay-ATI Obstruction Neck mobility What is DEFINITIVE airway? Tube in trachea BELOW vocal cords WHEN is surgical airway indicated? When you CANNOT do any other form of airway (2 attempts typically) What is contraindicated with suspicion of basilar skull frx? OPA, put in gastric tube for suction because it can go through up into cranium thru cribiform plate Peds kid with bicycle trauma is most likely goin to be? PANCREATIC injury Organ tear is more common with? Penetrating injury What type of pelvic fracture is binder indicated in 60 | P a g e HYPOXIA If you fail an IV in a kid what is next step? IO What is crystalloid infusion unit for kids 20/kg Two MCC of geriatric injuries? FALL, MVC A 22-year-old man is hypotensive and tachycardic after a shotgun wound to the left shoulder. His blood pressure is initially 80/40 mm Hg. After initial fluid resuscitation his blood pressure increases to 122/84 mm Hg. His heart rate is now 100 beats per minute and his respiratory rate is 28 breaths per minute. A tube thoracostomy is performed for decreased left chest breath sounds with the return of a small amount of blood and no air leak. After chest tube insertion, the most appropriate next step is: re-examine the chest A construction worker falls two stories from a building and sustains bilateral calcaneal fractures. In the emergency department, he is alert, vital signs are normal, and he is complaining of severe pain in both heels and his lower back. Lower extremity pulses are strong and there is no other deformity. The suspected diagnosis is most likely to be confirmed by: complete spine x-ray series Which of the following is true regarding the initial resuscitation of a trauma patient? Evidence of improved perfusion after fluid resuscitation could include improvement in Glasgow Coma Scale score on reevaluation. In managing a patient with a severe traumatic brain injury, the most important initial step is to: secure the airway A previously healthy, 70-kg (154-pound) man suffers an estimated acute blood loss of 2 liters. Which one of the following statements applies to this patient? An ABG would demonstrate a base deficit between -6 and -10 mEq/L. The physiological hypervolemia of pregnancy has clinical significance in the management of the severely injured, gravid woman by: increasing the volume of blood loss to produce maternal hypotension The best assessment of fluid resuscitation of the adult burn patient is: urinary output of 0.5 mL/kg/hr The diagnosis of shock must include: 61 | P a g e evidence of inadequate organ perfusion A 7-year-old boy is brought to the emergency department by his parents several minutes after he fell through a window. He is bleeding profusely from a 6-cm wound of his medial right thigh. Immediate management of the wound should consist of: direct pressure on the wound For the patient with severe traumatic brain injury, profound hypocarbia should be avoided to prevent: cerebral vasoconstriction with diminished perfusion After being involved in a motor vehicle crash, a 25-year-old man is brought to a hospital that has surgery capabilities available.. Computed tomography of the chest and abdomen shows an aortic injury and splenic laceration with free abdominal fluid. His blood pressure falls to 70 mm Hg after CT. The next step is: perform an exploratory laparotomy Which one of the following statements regarding abdominal trauma in the pregnant patient is TRUE? Leakage of amniotic fluid is an indication for hospital admission. The first maneuver to improve oxygenation after chest injury is: administer supplemental oxygen A 25-year-old man, injured in a motor vehicular crash, is admitted to the emergency department. His pupils react sluggishly and his eyes open to pressure. He does not follow commands, but he does moan periodically. His right arm is deformed and does not respond to pressure; however, his left hand reaches purposefully toward the stimulus. Both legs are stiffly extended. His GCS score is: 9 A 20-year-old woman who is at 32 weeks gestation, is stabbed in the upper right chest. In the emergency department, her blood pressure is 80/60 mm Hg. She is gasping for breath, extremely anxious, and yelling for help. Breath sounds are diminished in the right chest. The most appropriate first step is to: perform needle or finger decompression of the right chest Which one of the following findings in an adult is most likely to require immediate management during the primary survey? respiratory rate of 40 breaths per minute The most important, immediate step in the management of an open pneumothorax is: placement of an occlusive dressing over the wound The following are contraindications for tetanus toxoid administration: history of neurological reaction or severe hypersensitivity to the product 62 | P a g e A 56-year-old man is thrown violently against the steering wheel of his truck during a motor vehicle crash. On arrival in the emergency department he is diaphoretic and complaining of chest pain. His blood pressure is 60/40 mm Hg and his respiratory rate is 40 breaths per minute. Which of the following best differentiates cardiac tamponade from tension pneumothorax as the cause of his hypotension? breath sounds Bronchial intubation of the right or left mainstem bronchus can easily occur during infant endotracheal intubation because: The trachea is relatively short. A 23-year-old man sustains 4 stab wounds to the upper right chest during an altercation and is brought by ambulance to a hospital that has full surgical capabilities. His wounds are all above the nipple. He is endotracheally intubated, closed tube thoracostomy is performed, fluid resuscitation is initiated through 2 large-caliber IVs. FAST exam does not reveal intraabdominal injuries. His blood pressure now is 60/0 mm Hg, heart rate is 160 beats per minute, and respiratory rate is 14 breaths per minute (ventilated with 100% O2). 1500 mL of blood has drained from the right chest. The most appropriate next step in managing this patient is to: urgently transfer the patient to the operating room A 39-year-old man is admitted to the emergency department after an automobile collision. He is cyanotic, has insufficient respiratory effort, and has a GCS score of 6. His full beard makes it difficult to fit the oxygen facemask to his face. The most appropriate next step is to: restrict cervical motion and attempt orotracheal intubation using 2 people A patient is brought to the emergency department after a motor vehicle crash. He is conscious and there is no obvious external trauma. He arrives at the hospital completely immobilized on a long spine board. His blood pressure is 60/40 mm Hg and his heart rate is 70 beats per minute. His skin is warm. Which one of the following statements is TRUE? Flaccidity of the lower extremities and loss of deep tendon reflexes are expected. Which one of the following is the most effective method for initially treating frostbite? moist heat A 32-year-old man's right leg is trapped beneath his overturned car for nearly 2 hours before he is extricated. On arrival in the emergency department, his right lower extremity is cool, mottled, insensate, and motionless. Despite normal vital signs, pulses cannot be palpated below the right femoral artery and the muscles of the lower extremity are firm and hard. During the management of this patient, which of the following is most likely to improve the chances for limb salvage? surgical consultation for right lower extremity fasciotomy A patient arrives in the emergency department after being beaten about the head and face with a wooden club. He is comatose and has a palpable depressed skull fracture. His face is swollen and ecchymotic. He has gurgling respirations and vomitus on his face and clothing. The most appropriate step after providing supplemental oxygen and elevating his jaw is to: 65 | P a g e - Obtain ABG analysis - Transesophageal echocardiography A construction worker falls two stories from a building and sustains bilateral calcaneal fractures. In the emergency department, he is alert, vital signs are normal, and he is complaining of severe pain in both heels and his lower back. Lower extremity pulses are strong and there is no other deformity. The suspected diagnosis is most likely to be confirmed by: - complete spine x-ray series Which of the following is true regarding the initial resuscitation of a trauma patient? - A patient with a torso gunshot wound and hypotension should receive crystalloid fluid resuscitation until the blood pressure is normal - Evidence of improved perfusion after fluid resuscitation could include improvement in Glasgow Coma Scale score on reevaluation - Massive transfusion is defined as transfusion of more than 10 units of packed red blood cells and plasma within 24 hours - When tranexamic acid is administered by pre-hospital providers, a second dose is required within 24 hours - Fluid resuscitation is far more important than bleeding control in trauma patient In managing a patient with a severe traumatic brain injury, the most important initial step is to: - Secure the airway A previously healthy, 70-kg (154-pound) man suffers an estimated acute blood loss of 2 liters. Which one of the following statements applies to this patient? - His pulse pressure will be widened. - His urinary output will be at the lower limits of normal. xxx - He will have tachycardia, but no change in his systolic blood pressure. - An ABG would demonstrate a base deficit between -6 and -10 mEq/L. - His systolic blood pressure will be maintained with an elevated diastolic pressure. The physiological hypervolemia of pregnancy has clinical significance in the management of the severely injured, gravid woman by: - increasing the volume of blood loss to produce maternal hypotension. The best assessment of fluid resuscitation of the adult burn patient is: - urinary output of 0.5 mL/kg/hr The diagnosis of shock must include: - evidence of inadequate organ perfusion A 7-year-old boy is brought to the emergency department by his parents several minutes after he fell through a window. He is bleeding profusely from a 6-cm wound of his medial right thigh. Immediate management of the wound should consist of: 66 | P a g e - direct pressure on the wound For the patient with severe traumatic brain injury, profound hypocarbia should be avoided to prevent: - respiratory acidosis xxx - metabolic acidosis - Cerebral vasoconstriction with diminished perfusion - neurogenic pulmonary edema - shift of the oxyhemoglobin dissociation curve After being involved in a motor vehicle crash, a 25-year-old man is brought to a hospital that has surgery capabilities available. Computed tomography of the chest and abdomen shows an aortic injury and splenic laceration with free abdominal fluid. His blood pressure falls to 70 mm Hg after CT. The next step is: - perform an exploratory laparotomy Which one of the following statements regarding abdominal trauma in the pregnant patient is TRUE? - The fetus is in jeopardy only with major abdominal trauma. - Leakage of amniotic fluid is an indication for hospital admission. - Indications for peritoneal lavage are different from those in the nonpregnant patient. - With penetrating trauma, injury to the mother's abdominal hollow viscus is more common in late than in early pregnancy. - The secondary survey follows a different pattern from that of the nonpregnant patient. The first maneuver to improve oxygenation after chest injury is: - Administer supplemental oxygen - intubate the patient - assess arterial blood gases - ascertain the need for a chest tube - obtain a chest x-ray A 25-year-old man, injured in a motor vehicular crash, is admitted to the emergency department. His pupils react sluggishly and his eyes open to pressure. He does not follow commands, but he does moan periodically. His right arm is deformed and does not respond to pressure; however, his left hand reaches purposefully toward the stimulus. Both legs are stiffly extended. His GCS score is: 9 A 20-year-old woman who is at 32 weeks gestation, is stabbed in the upper right chest. In the emergency department, her blood pressure is 80/60 mm Hg. She is gasping for breath, extremely anxious, and yelling for help. Breath sounds are diminished in the right chest. The most appropriate first step is to: - perform needle or finger decompression of the right chest Which one of the following findings in an adult is most likely to require immediate management during the primary survey? 67 | P a g e - distended abdomen - Glasgow Coma Scale score of 11 - temperature of 36.5°C (97.8°F) - deforming of the right thigh - respiratory rate of 40 breaths per minute The most important, immediate step in the management of an open pneumothorax is: - placement of an occlusive dressing over the wound The following are contraindications for tetanus toxoid administration: - history of neurological reaction or severe hypersensitivity to the product - local side effects - muscular spasms - pregnancy - all of the above xxx A 56-year-old man is thrown violently against the steering wheel of his truck during a motor vehicle crash. On arrival in the emergency department he is diaphoretic and complaining of chest pain. His blood pressure is 60/40 mm Hg and his respiratory rate is 40 breaths per minute. Which of the following best differentiates cardiac tamponade from tension pneumothorax as the cause of his hypotension? - breath sounds Bronchial intubation of the right or left mainstem bronchus can easily occur during infant endotracheal intubation because: - The trachea is relatively short. A 23-year-old man sustains 4 stab wounds to the upper right chest during an altercation and is brought by ambulance to a hospital that has full surgical capabilities. His wounds are all above the nipple. He is endotracheally intubated, closed tube thoracostomy is performed, fluid resuscitation is initiated through 2 large-caliber IVs. FAST exam does not reveal intraabdominal injuries. His blood pressure now is 60/0 mm Hg, heart rate is 160 beats per minute, and respiratory rate is 14 breaths per minute (ventilated with 100% O2). 1500 mL of blood has drained from the right chest. The most appropriate next step in managing this patient is to: - urgently transfer the patient to the operating room A 39-year-old man is admitted to the emergency department after an automobile collision. He is cyanotic, has insufficient respiratory effort, and has a GCS score of 6. His full beard makes it difficult to fit the oxygen facemask to his face. The most appropriate next step is to: - restrict cervical motion and attempt orotracheal intubation using 2 people A patient is brought to the emergency department after a motor vehicle crash. He is conscious and there is no obvious external trauma. He arrives at the hospital completely immobilized on a long spine board. His blood pressure is 60/40 mm Hg and his heart rate is 70 beats per minute. His skin is warm. Which one of the following statements is TRUE? 70 | P a g e A 40-year-old woman who was a restrained driver in a motor vehicle crash is evaluated in the emergency department. She is hemodynamically normal and found to be paraplegic at the level of T10. Which of the following are true regarding her evaluation and management? - Neurogenic shock is likely to develop. - Imaging of her complete spine is required prior to transfer to a trauma center. - Given the injury level knee extension would be expected. - Log rolling using 4 people is a safe approach to restrict spinal motion when moving her. Presence of bulbocarvenous reflex indicates a better prognosis. A trauma patient presents to your emergency department with inspiratory stridor and a suspected c- spine injury. Oxygen saturation is 88% on high-flow oxygen via a nonrebreathing mask. The most appropriate next step is to: - restrict cervical motion and establish a definitive airway When applying the Rule of Nines to infants... - The head is proportionally larger in infants than in adults. A healthy young male is brought to the emergency department following a motor vehicle crash. His vital signs are a blood pressure of 84/60, pulse 123, GCS 10. The patient moans when his pelvis is palpated. After initiating fluid resuscitation, the next step in management is: - placement of a pelvic binder - transfer to a trauma center - pelvic x-ray - insert urinary catheter - repeat examination of pelvis Which one of the following situations requires Rh immunoglobulin administration to an injured woman? - negative pregnancy test, Rh negative, and has torso trauma - positive pregnancy test, Rh positive, and has torso trauma - positive pregnancy test, Rh negative, and has torso trauma - positive pregnancy test, Rh positive, and has an isolated wrist fracture - positive pregnancy test, Rh negative, and has an isolated wrist fracture A 22-year-old female athlete is stabbed in her left chest at the third interspace in the anterior axillary line. On admission to the emergency department and 15 minutes after the incident, she is awake and alert. Her heart rate is 100 beats per minute, blood pressure 80/60 mm Hg, and respiratory rate 20 breaths per minute. A chest x-ray reveals a large left hemothorax. A left chest tube is placed with an immediate return of 1600 mL of blood. The next management step for this patient is: - prepare for an exploratory thoracotomy 71 | P a g e A 6-year-old boy walking across the street is struck by the front bumper of a sports utility vehicle traveling at 32 kph (20 mph). Which one of the following statements is TRUE about this patient? - A flail chest is probable. - A symptomatic cardiac contusion is expected. - A pulmonary contusion may be present in the absence of rib fractures. - Transection of the thoracic aorta is more likely than in an adult patient. - Rib fractures are commonly found in children with this mechanism of injury. ABCDE Mnemonic A - Airway with cervical spine protection B - Breathing C - Circulation, stop bleeding D - Disability or neurological status E - Exposure (undress) & Environment (temperature control) Breathing & ventilation - injuries that impair ventilation Severe impairment: * Tension pneumothorax * Flail chest with pulmonary contusion * Massive haemothorax * Open pneumothorax Lesser impairment * Simple pneumothorax / haemothorax * Rib # * Pulmonary contusion Circulation with heamorrhage control - assessing haemodynamic status LEVEL OF CONSCIOUSNESS Altered level of consciousness may indicate: - ↓ circulating blood volume ∴ ↓ cerebral perfusion SKIN COLOR PULSE * Full, regular pulse = normovolaemia * Rapid, thready pulse ?hypovolaemia Adjuncts to primary survey - ECG Indicators of blunt cardiac injury: DYSRHYTHMIAS: * Tachycardia * AF * PVC * ST changes PEA can indicate: 72 | P a g e * Cardiac tamponade * Tension pneumothorax * Hypovolaemia Consider hypoxia & hypo-perfusion if: * Bradycardia * Aberrant conduction * Premature beats Secondary survey Once primary survey (ABCDE) is complete, resuscitation underway & vital functions normalisation has been demonstrated WHAT IS IT? - Head to toe evaluation - History Secondary survey - AMPLE history A - Allergies M - Medications P - Past illnesses - Pregnancy L - Last meal E - Events - Environment related to injury Blunt trauma - Seat belt - Steering wheel deformation - Damage to car + intrusion - Ejection from vehicle Secondary survey - Physical examination - HEAD SCALP & HEAD: - Lacerations / contusions / fractures EYES: - Visual acuity - Pupillary size - Haemorrhage - conjunctiva / fundus - Penetrating injury - Contact lenses - Lens dislocation - Ocular entrapment - assess eye movements MAXILLOFACIAL STRUCTURES: i) Palpate bony structures ii) Assess occlusion 75 | P a g e vi) Anatomical variations - Receding chin - Overbite - Short, muscular neck LEMON assessment of difficult intubation L = LOOK EXTERNALLY ?Characteristic known to cause difficult intubation / ventilation - Small mouth or jaw / Large overbite / facial trauma EVALUATE 3-3-2 RULE To allow alignment of pharyngeal, laryngeal & oral axes, following relationship must be observed a) 3 fingers breadth distance between incisor teeth b) 3 finger breadth distance between hyoid bone & chin c) 2 finger breadth distance between thyroid notch & floor of mouth M = MALLAMPATI Visualise hypopharynx Mallampati classification Pt upright or supine - open mouth & protrude tongue CLASSIFICATIONS: Class I Soft palate, uvula, fauces, pillars visible Class II Soft palate, uvula, fauces visible Class III Soft palate, base of uvula visible Class IV Hard palate only visible Anatomy of hypopharynx FAUCES - Aperture by which mouth communicates with pharynx Boundaries: Superior = soft palate Inferior = dorsum of tongue Lateral = glossopalatine arches PILLARS Anterior pillar = glossopalatine arch Posterior pillar = pharyngopalatine arch PALATINE TONSILS Masses between glossopalatine & pharyngopalatine arches 76 | P a g e Airway maintenance techniques 1) Chin-lift 2) Jaw thrust 3) Oropharyngeal airway - Do not use 180° rotation method for insertion in children as it can damage mouth & pharynx 4) Nasopharyngeal airway 5) Extraglottic & supraglottic devices i) LMA / Intubating LMA ii) Laryngeal tube airway iii) Multilumen oesophageal airway Definitive airway Tube placed in trachea with cuff inflated below the vocal cords 3 types: i) Orotracheal tubes ii) Nasotracheal tubes iii) Surgical airway Indications for definitive airway NEED FOR AIRWAY PROTECTION 1) Severe maxillofacial fractures 2) Risk of obstruction - Neck haematoma - Laryngeal / tracheal injury - Stridor 3) Risk for aspiration - Bleeding - Vomitng 4) Unconscious NEED FOR VENTILATION 1) Inadequate respiratory effort - Tachypnoea - Hypoxia - Hypercarbia - Cyanosis 2) Massive blood loss & need for volume resusicitation 3) Severe closed head injury + need for hyperventilation if neurological deterioration occurs 4) Apnoea 77 | P a g e - NM paralysis - Unconscious Contraindications to nasotracheal intubation Fractures - facial / frontal sinus / basilar skull / cribiform plate Signs: i) Nasal fracture ii) Raccoon eyes (bilateral periorbital ecchymosis) iii) Battle's sign (post-auricular ecchymosis) iv) Rhinorrhoea / otorrhoea (CSF leakage) Suxamethonium / succinylcholine - caution Depolarising NM blocker CAUTION i) Hyperkalaemia - Caution with severe crush injuries / burns / electrical injuries ii) Malignant hyperthermia → uncontrolled skeletal muscle oxidative metabolism Surgical airway Surgical airway required when airway obstructed by: - Oedema of glottis - Laryngeal fracture - Oropharyngeal haemorrhage Surgical cricothyroidotomy preferred to tracheostomy: i) Easier to perform ii) Less bleeding iii) Requires less time to perform Needle cricothyroidotomy a) Plastic cannula through cricothyroid membrane b) Connected to 15L O2 c) Jet insufflation - 1sec ON / 4 sec OFF ** Inadequate exhalation ∴ CO2 accumulates ** Can be used for 30-45 mins Assessing adequate oxygenation PULSE OXIMETRY - Measures O2 sats of arterial blood - Does not measure PaO2 ** Sats ≥95% suggests adequate peripheral arterial oxygentaion (PaO2 >70mmHg or 9.3kPa) NOTE: i) Requires intact peripheral perfusion ii) Cannot distinguish oxyhaemoglobin from carboxyhaemoglobin / metheamoglobin LIMITED USEFULNESS IN: 80 | P a g e a) Venous capacitance b) Volume status c) Difference between mean venous systemic pressure & right atrial pressure * Pressure differential determines venous flow * Basic cardiac physiology - Preload 2 Venous system like a reservoir / capacitance system Volume of blood divided into 2 components: i) Volume of blood remaining in system if pressure in system was zero - Does not contribute to mean systemic venous pressure ii) Venous volume that contributes to mean systemic venous pressure * Pressure gradient drives venous flow * Basic cardiac physiology - myocardial contractility Volume of venous return determines myocardial muscle fibre length after ventricular filling at the end of diastole - Muscle fibre length relates to muscle contractility according to Starling's law Frank-Starling law of the heart Stroke volume increases in response to increase in volume of blood filling the heart (end diastolic volume) when all other factors remain constant Basic cardiac physiology - Aferload Resistance to forward flow of the heart Blood loss pathophysiology CIRCULATORY RESPONSES TO BLOOD LOSS Compensation: - Vasoconstriction of cutaneous / muscle / visceral circulation - Preserves blood flow to kidneys / heart / brain → TACHYCARDIA = earliest sign ↑ PERIPHERAL VASCULAR RESISTANCE - ↑ DBP & ↓ PULSE PRESSURE Recognition of shock Signs of shock: i) Tachycardia ii) Cutaneous vasoconstriction iii) ↓ pulse pressure iv) ↑ RR v) ↓SBP 81 | P a g e Neurogenic shock * Isolated intracranial injuries do not cause shock * - Cervical / upper thoracic spinal cord injury → hypotension due to loss of sympathetic tone Haemorrhagic shock Haemorrhage = acute loss of circulating blood volume * Normal adult blood volume ~ 7% of body weight * Class I haemorrhage (≤ 15 blood volume loss) MINIMAL CLINIC SYMPTOMS i) Possibly minimal tachycardia No changes in BP / PP / RR - For healthy patients, no replacement required ∵ cap. refill & other compensatory mechanisms wiil restore blood volume within 24 hrs Class II haemorrhage (15-30% blood volume loss) ~ 750-1500mL blood loss (70kg man) i) Tachycardia (>100bpm) ii) Tachypnoea iii) ↓ pulse pressure iv) Subtle CNS changes - anxiety / fright / hostility v) UO only mildly affected (20-30 mL/hr) Most pts stabilised with crystalloid replacement Class III haemorrhage (30-40% blood loss) ~ 1500-2000mL (70kg man) Classic signs of inadequate perfusion: i) Tachycardia ii) Tachypnoea iii) Changes in mental status iv) ↓ SBP - Require transfusion 82 | P a g e Class IV haemorrhage (>40% blood loss) Immediately life-threatening i) Tachycardia ii) ↓ SBP iii) Very narrow pulse pressure iv) Negligible UO v) Depressed mental status vi) Cold, pale skin Contraindication for transurethral catheterisation (male) i) Blood at urethral meatus ii) High-riding, mobile or non-palpable prostate Need radiographic confirmation of intact urethra Vascular access 2 large-bore cannula * Rate of flow is: a) Proportional to fourth power of radius of cannula b) Inversely proportional to length ∴ wide, short cannulas are preferred for rapid infusion of large volumes of fluid Composition of commonly used IV fluids Evaluation of fluid resuscitation & organ perfusion - Acid-base balance Early hypovolaemic shock: - Tachypnoea → respiratory alkalosis - Followed by mild metabolic acidosis Long-standing / severe shock: - Severe metabolic acidosis - Inadequate tissue perfusion → anaerobic metabolism → lactic acid production Response to initial fluid resuscitation Pattern of response to initial fluid administration, 3 groups: i) Rapid response ii) Transient response iii) Minimal / no response 85 | P a g e iv) AV fistula v) Chylothorax vi) Infection vii) Air embolism Complication of intraosseous puncture i) Infection ii) Through & through bone penetration iii) Subcutaneous & subperiosteal infiltration iv) Skin pressure necrosis v) Physeal plate injury vi) Haematoma Venous cutdown Sites for venous cutdown: 1) Greater saphenous vein (medial ankle) - 2cm anterior & superior to medial malleolus 2) Antecubital medial basilic vein - 2.5cm lateral to medial epicondyle of humerus at flexion crease of elbow Complications of peripheral venous cutdown i) Cellulitis ii) Haematoma iii) Phlebitis iv) Perforation of posterior vein wall v) Venous thrombosis vi) Nerve transection vii) Arterial transection Beck's triad 3 medical signs associated with cardiac tamponade: i) Elevated venous pressure - ↓ diastolic filling of RV due to expanding pericardial sac → ↑CVP & distended neck veins ii) Low arterial BP - Pericardial fluid accumulation impairs ventricular stretch & SV iii) Muffled heart sounds Kussmaul's sign Paradoxical rise in JVP on inspiration - indicates limited RV filling due to right heart failure PATHOPHYSIOLOGY Normally: JVP falls on inspiration - ↓ pressure as thorax expands & ↑ RV expansion during diastole Kussmaul sign: 86 | P a g e RV filling impaired due to: i) Pericardial fluid ii) Poor compliance of myocardium / pericardium - Impaired filling → reduced venous return & jugular venous distension Haemothorax - guideline for operative exploration i) 1500mL blood obtained immediately from chest tube ii) >200mL/hr for 2-4 hrs is drained iii) Blood transfusion required Complications of needle thoracocentesis i) Local haematoma ii) Pneumothorax iii) Lung laceration Complications of chest tube insertion i) Laceration / puncture of thoracic / abdominal organs ii) Pleural infection iii) Damage to neurovascular bundle iv) Incorrect tube position v) Chest tube occlusion, kinking vi) Persistent penumothorax - leak vii) Subcutaneous emphysema viii) Recurrence of pneumothorax ix) Lung fails to expand - plugged bronchus x) Anaphylactic / allergic reaction to surgical preparation / anaesthetic Complications of pericardiocentesis i) Aspiration of ventricular blood instead of pericardial blood ii) Laceration of ventricular epicardium / myocardium iii) Coronary artery / vein laceration iv) New haemopericardium - secondary to laceration of cardiac structure v) VF vi) Pneumothorax vii) Great vessel puncture → worsening pericardial tamponade viii) Oesophageal puncture → mediastinitis ix) Peritoneal puncture → peritonitis / false positive aspirate Anatomy of abdomen ANTERIOR ABDOMEN (boundaries) Superior - between costal margins Inferior - Inguinal ligaments & symphysis pubis THORACO-ABDOMEN (boundaries) Area inferior to: 87 | P a g e i) Trans-nipple line anteriorly ii) Infra-scapular line posteriorly + Superior to costal margins FLANK Area between ANTERIOR & POSTERIOR AXILLARY LINES from SIXTH ICS to the ILIAC CREST BACK - Posterior to posterior axillary line - Tip of scapulae to iliac crests Truncal & cervical injuries from restraint devices - Lap Seat Belt: - Compression - Hyperflexion - Tear / avulsion of mesentery (Bucket Handle) - Rupture of small bowel / colon - Thrombosis if iliac artery or abdominal aorta - Chance failure of lumbar vertebrae - Pancreatic / duodenal injury Truncal & cervical injuries from restraint devices - Shoulder Harness: - Sliding under seat belt (submarining) - Compression - Intimal tear / thrombosis in innominate, carotid, subclavian or vertebral arteries - Fracture / dislocation of cervial spine - Pulmonary contusion - Rupture of upper abdominal viscera Truncal & cervical injuries from restraint devices - Air Bag: - Contact - Contact / deceleration - Flexion (unrestrained) - Hyperextension (unrestrained) - Corneal abrasions - Face, neck, chest abrasions - Cardiac rupture - Cervical spine - Thoracic spine fracture Diagnostic peritoneal lavage RELATIVE CONTRAINDICATIONS i) Previous abdominal operations ii) Morbid obesity iii) Advanced cirrhosis iv) Coagulopathy --- OPEN / CLOSED (seldinger) INFRAUMBILICAL TECHNIQUE 90 | P a g e MINOR TRAUMATIC BRAIN INJURY (MTBI) - H/O disorientation, amnesia, transient LOC & pt conscious & talking - Most make uneventful recoveries - Determine amnesia before (retrograde) & after (anterograde) incident Management of moderate brain injury (GCS 9-12) Most: - Able to follow simple commands May have: - confusion / somnolent - focal neurology (e.g. hemiparesis) 10-20% deteriorate & lapse into coma * Obtain CT head * Priorities for initial evaluation & triage of patients with severe brain injuries 1) All comatose pts → resuscitation (ABCDEs) on arrival 2) When BP normalized, perform neuro exam (GCS & pupils) 3) If SBP cannot be brought to >100mmHg - establish cause 4) If SBP > 100mmHg post-resuscitation & there is evidence of intrracranial mass (unequal pupils, focal neurology) → CT head Head injury - primary survey & resuscitation 1 Brain injury adversely affected by secondary insults Mortality for severe brain injury + hypotension is double that with no hypotension ∴ achieve cardiopulmonary stabilization Head injury - primary survey & resuscitation 2 AIRWAY & BREATHING Transient resp. arrest & hypoxia = common - could cause secondary brain injury * Set ventilation parameters to maintain PCO2 ~35mmHg / 4.7 kPa Head injury - primary survey & resuscitation 3 CIRCULATION Hypotension not usually due to brain injury EXCEPT - medullary failure - spinal cord injury * Establish euvolaemia ASAP * - Neuro exam in hypotension can be unreliable * Head injury - primary survey & resuscitation 4 91 | P a g e FOCUSED NEUROLOGICAL EXAM 1) GCS 2) Pupils 3) Focal neurology Head injury - secondary survey Serial neuro exams - detect deterioration - Sign of temporal lobe (uncal) herniation = pupil dilation & loss of response Medical therapies for brain injury Primary aim = prevent secondary damage i) IV fluids ii) Temporary hyperventilation iii) Mannitol iv) Hypertonic saline v) Barbiturates vi) Anticonvulsants Medical therapies for brain injury - IV fluids Resuscitate pt to restore normovolaemia - Do not use hypotonic fluids or glucose-containing fluids - Hyperglycaemia = harmful for brain - Avoid hyponatraemia → cerebral oedema Medical therapies for brain injury - Hyperventilation Normocarbia is preferred HYPERVENTILATION - ↓ PaCO2 → cerebral vasoconstriction - Preferable to keep PaCO2 ~ 35mmHg (4.7kPa) Medical therapies for brain injury - Mannitol ↓ ICP OSMOTHERAPY - ↑ serum osmolality & shifts fluid from intracellular to intravascular compartment AVOID IN: - Hypotension (Mannitol does not lower ICP in hypovolaemia & is a osmotic diuretic) INDICATION Acute neurological deterioration in in euvolaemic pt DOSE 20% solution (20g of mannitol per 100ml solution) Bolus (1g/kg) given rapidly (over 5 mins) Medical therapies for brain injury - Hypertonic saline 92 | P a g e Osmotherapy ↓ ICP - Concentrations 3-23.4% used - Does not act as diuretic - Does not work in hypovolaemia Brain death - diagnosis i) GCS = 3 ii) Nonreactive pupils iii) No brainstem reflexes (oculocephalic, corneal, Doll's eyes, gag reflex) iv) No spontaneous ventilatory effort ANCILLARY STUDIES a) EEG b) Cerebral blood flow (CBF) studies c) Cerebral angiography Clinical assessment of spinal cord tracts CORTICOSPINAL TRACT Function - controls ipsilateral motor power Testing - Muscle strength SPINOTHALAMIC TRACT Function - transmits contralateral pain & temperature Testing - Pinprick & light touch DORSAL COLUMNS Function - Ipsilateral proprioception, vibration & light touch Testing - Proprioception & vibration Sensory examination KEY SENSORY LANDMARKS (American Spinal Injury Association, ASIA) C2 - Occipital protuberance C3 - Supraclavicular fossa C4 - Top of AC joint C5 - Lateral antecubital fossa C6 - Thumb, dorsal surface, proximal phalanges C7 - Middle finger, dorsal surface, proximal phalanges C8 - Little finger, dorsal surface, proximal phalanges T1 - Medial (ulnar) antecubital fossa T2 - Apex of axilla 95 | P a g e Odontoid fractures 60% of C2 fractures involve odontoid process TYPE 1 Tip of odontoid - uncommon TYPE 2 Base of dens - most common TYPE 3 Base of dens & extend obliquely into body of axis Fractures & dislocations (C3 - C7) C3 FRACTURE - Uncommon - Positioned between vulnerable axis & mobile C5/6 (responsible for greatest flexion / extension) MOST COMMON LEVEL OF FRACTURE C5 MOST COMMON LEVEL OF SUBLUXATION C5 on C6 Thoracic Spine Fractures (T1 - T10) 1) Anterior wedge compression injuries - axial loading with flexion 2) Burst injury - vertical-axial compression 3) Chance fractures - compression injury to anterior vertebral body + transverse fracture through the posterior vertebral body. - violent forward flexion 4) Fracture - dislocations - extreme flexion STABLE simple compression fracture UNSTABLE Burst / Chance / Fracture dislocation Primary survey - Differentiate hypovolaemic vs. neurogenic shock HYPOVOLAEMIC i) ↓ BP ii) ↑ HR 96 | P a g e iii) Cool extremities NEUROGENIC i) ↓ BP ii) ↓ HR iii) Warm extremities Injuries associated with MSK injuries: Clavicular / scapular fracture Fracture and/or dislocation of shoulder Missed / associated injury: Major thoracic injury, esp pulmonary contusion & rib fractures Injuries associated with MSK injuries: Displaced thoracic spine fracture Missed / associated injury: Thoracic aortic rupture Injuries associated with MSK injuries: Spine fracture Missed / associated injury: Intrabdominal injury Injuries associated with MSK injuries: Fracture/dislocation of elbow Missed / associated injury: - Brachial artery injury - Median, ulnar, radial nerve injury Injuries associated with MSK injuries: Femur fracture Missed / associated injury: - Femoral neck fracture - Posterior hip dislocation Injuries associated with MSK injuries: Posterior knee dislocation Missed / associated injury: - Femoral fracture - Posterior hip dislocation Injuries associated with MSK injuries: - Knee dislocation - Displaced tibial plateau fracture 97 | P a g e Missed / associated injury: Popliteal artery & nerve injuries Injuries associated with MSK injuries: Calcaneal fracture Missed / associated injury: - Spine injury or fracture - Fracture-dislocation of hind foot - Tibial plateau fracture Injuries associated with MSK injuries: Open fracture Missed / associated injury: 70% incidence of associated nonskeletal injury Factors that increase risk for upper airway obstruction i) Increasing burn size & depth ii) Burns to head & face iii) Inhalation injury iv) Burns inside mouth How to identify inhalation injury Clinical indications of inhalation injury: - Face and/or neck burns - Eyebrow / nasal vibrissae singeing - Carbon deposits in mouth / nose - Carbonaceous sputum - Acute inflammation of oropharynx (incl. erythema) - Hoarseness - H/O impaired mentation and / or confinement in burning environment - Explosion with burns to head / torso - CarboxyHb >10% in pt involved in fire Rule of nines - adult Rule of nines - children